HELP ME PLEASE!!!! URGENT!!!

HELP ME PLEASE!!!! URGENT!!!

Answers

Answer 1

Answer:

Step-by-step explanation:

f(x)=4x+1

g(x) = x^2-5

(f*g)(x) = 4x^3+x^2-20x-5


Related Questions

PLS HELP

Simplify the function f(x) =
3x
(81)
4
Then determine the key aspects of the function.
The initial value is
The simplified base is
The domain is
The range is

Answers

Answer:

Step-by-step explanation:

This follows the form

[tex]y=a(b)^x[/tex]

where a is the initial value and b is the base with the exponent.  Using that information, we can see that the initial value in our function is 1/3. Simplifying the base will take some work. Let's first rewrite this is a radical:

[tex]81^{\frac{3x}{4} }=\sqrt[4]{81^{3x} }[/tex]

Now let's break up 81 into its factors. 81 is 9*9 which is 3*3*3*3.  Therefore,

[tex]81=3^4[/tex]

We will use that as a simplification:

[tex]\sqrt[4]{(3^4)^{3x}}[/tex]  which simplifies to

[tex]\sqrt[4]{3^{12x}}[/tex]

Rewriting that as an exponent looks like this:

[tex]3^{\frac{12x}{4}}[/tex] which simplifies to

[tex]3^{3x}[/tex]

That's the answer for the second part.  The whole exponential equation now is

[tex]f(x)=\frac{1}{3}(3)^{3x}[/tex]

The domain for an exponential function is all real numbers and the range is

y > 0

Answer:

1/3

27

all real numbers

y > 0

ON EDGE

Step-by-step explanation:

Can someone please help

Answers

Answer:

The numbers to input are 2, 2 , 7

Step-by-step explanation:

252 = 2² * 3² * 7

So the numbers to input are 2, 2 , 7

En un programa de televisión se hacen 30 preguntas. Por cada respuesta correcta se suman 8 puntos, por cada respuesta errónea se restan 5 puntos y por aquellas preguntas que no se contesten no se suman ni se restan puntos. Si un participantes obtuvo 13 puntos, ¿Cuantas respuestas erróneas pudo tener?

Answers

Answer:

7 wrong answers.

Step-by-step explanation:

We know that when you answer a question well, you earn 8 points and a bad one loses 5 points, this means that when you answer 1 good and 1 bad, there is a total of 3 points (8-5).

This means that when answering 5 good and 5 bad, in total it would be 3 * 5 points, that is to say 15 points, that is, it goes through 2 points. To subtract 2 points, you would have to answer 1 good and 2 bad, (8 - 2 * 5), which turns out to be -2 points.

In total there would be 6 questions right and 7 questions wrong, like this:

8 * 6 + 5 * 7 = 13

13 points, in 13 questions (6 correct + 7 incorrect)

The rest of the questions were not answered so that the score does not go up or down.

Which means you got 7 wrong answers.

Find the value of a . A.18 B.21 C.20 D.17

Answers

Answer:

a =18

Step-by-step explanation:

The two angles are vertical angles and vertical angles are equal

6a +11 = 2a+83

Subtract 2a from each side

6a-2a +11 = 2a-2a+83

4a +11 =83

Subtract 11 from each side

4a +11 -11 = 83-11

4a = 72

divide each side by 4

4a/4 = 72/4

a =18

How many digits will be in the quotient?
39 4,641

Answers

Answer:

the answer is 119 so three digits

Step-by-step explanation:

Answer:

3 digits

Step-by-step explanation:

4641/39=119

Identify the like terms in the expression.

Answers

Answer:D

The answer is D.

Step-by-step explanation:

Like terms are only related to the variable at the end, and the variable also has to have the same exponent or it is not a like term.

Find the sum of 6(1-a^2), -2(3-a+2a^2) and 5(-2a+3a^2)​

Answers

Answer:

[tex]5a^2-8a[/tex]

Step-by-step explanation:

[tex]6(1-a^2)-2(3-a+2a^2)+5(-2a+3a^2)[/tex]

[tex](6)(1)+(6)(-a^2)+(-2)(3)+(-2)(-a)+(-2)(2a^2)+(5)(-2a)+(5)(3a^2)[/tex]

[tex]6+-6a^2+-6+2a+-4a^2+-10a+15a^2[/tex]

[tex](-6a^2+-4a^2+15a^2)+(2a+-10a)+(6+-6)[/tex]

[tex]5a^2-8a[/tex]

Please help! Correct answer only, please! I need to finish this assgnment this week! Matrix P has a dimensions of 3 X 4 and Matrix Q has the dimensions 4 X 5. Determine the dimensions of the matrix PQ if it is possible. Explain why if it is not. (refer to video part 4) Group of answer choices A. Matrix PQ would have the dimensions 3 X 4 B. Matrix PQ would have the dimensions 4 X 5 C. Matrix PQ would have the dimensions 3 X 5 D. These matrices can not be multiplied because their dimension don't align.

Answers

Answer: C

Step-by-step explanation:

Your answer is correct! When you multiply 2 matrices, the inner dimensions have to be equal. If they are equal, the resulting dimensions would be the outer dimentions.

(3×4)(4×5)

Since the inner (bolded) dimensions are the same, in this case 4×4, we know these matrices can be multiplied.

Since these matrices can be multiplied, the product will be 3×5, the underlined outer dimensions.

HELPPPPPP ITS ABOUT EQUATIONSSSS HELPPPPP Explanation needed HELPPPPP PLEASEE THIS IS THE LAST QUESTION LEFTTTTTTT

Answers

Answer:

C. the average total cost for the first month of a gym membership

Step-by-step explanation:

x=1 is for month 1,

the value of y includes 1-month fee and one off payment, so this is the average total for the first month of membership

Answer:

It is the average total cost for the first month of a gym membership

Step-by-step explanation:

y = 34.99x+49

The 49 is the cost to join the gym and the 34.99 is the monthly cost

Let x =1 which is the cost after one month

It includes the cost to join and the 1st month membership

It is the average total cost for the first month of a gym membership

Which angle in ADEF has the largest measure?

Answers

Answer:

F is the largest angle

Step-by-step explanation:

The largest angle is opposite the largest side.  The smallest angle is opposite the smallest side.

The largest side is 4 so the largest angle is F

Jasper decided to save $100 at the end of each month for a year and deposit it in a bank account that earns an annual interest rate of 0.3%, compounded monthly. Use the formula for an annuity, F, to determine how much money will be in the account at the end of the 6th month, rounding your answer to the nearest penny.
Note: Your interest rate must be converted to a decimal

Answers

Answer: 600.38

Step-by-step explanation:

just put in the answer you lazy

(6) Work out
5 1
6 12

Answers

Answer:

=123

Step-by-step explanation:

k (t) = 10t - 19
k(-7) =

Answers

Answer:

k(-7) = - 89

Step-by-step explanation:

k (t) = 10t - 19

Let t = -7

k(-7) =10 * -7 - 19

        =-70-19

        -89

Answer:

[tex]k (-7) = -89[/tex]

Step-by-step explanation:

[tex]k (t) = 10t - 19[/tex]

[tex]k (-7) = 10(-7) - 19[/tex]

[tex]k (-7) = -70 - 19[/tex]

[tex]k (-7) = -89[/tex]

Please help me!!!!!!

Answers

Answer:

C

Step-by-step explanation:

The addition of the x^5 term makes it non proportional

Answer:

C.

Step-by-step explanation:

A. y = (6x + 3) - 3= 6x

B. y = - 15x

D. = - 1/3 x

All of these are proportional because they have general formula y = kx.

The circumference of the earth is given. 


Circumference of earth: 24,901 miles


What is the diameter of earth? Round your answer to the nearest tenth. Use 3.14 for π.


Answers

Answer:

7930.3 miles = d

Step-by-step explanation:

The circumference equals

C = pi *d

24901 = 3.14 d

Divide each side by 3.14

24901 / 3.14 = d

7930.254777 = d

Rounding to the nearest tenth

7930.3 =d

You are going to use an incline plane to lift a heavy object to the top of a shelving unit with a height of 5 ft. The base of the incline plane is 15 ft from the shelving unit. What is the length of the incline​ plane?

Answers

Answer:

15.8 ft

Step-by-step explanation:

The inclined plane, the base of and the shelving unit form the shape of a right angled triangle.

The hypotenuse is the length of the inclined plane, h.

The base of the triangle is 15 ft.

The height of the triangle is 5 ft.

To find the hypotenuse, h, we have to use Pythagoras rule:

[tex]h^2 = a^2 + b^2[/tex]

where a = height of the triangle

b = base of the triangle

Therefore:

[tex]h^2 = 5^2 + 15^2\\\\h^2 = 25 + 225 = 250\\\\h = \sqrt{250}\\ \\h = 15.8 ft[/tex]

The inclined plane is 15.8 ft long.

Robert gets a loan from his bank. He agrees to borrow £6,000 at a fixed annual simple interest rate of 7%. He also agrees to pay the loan back over a 10-year period. How much money in total will he have paid back at the end of the 10 years?​

Answers

Answer:

The total amount repayable will be £8,280.24.

Step-by-step explanation:

Use the attached formula

Solve for m.

3>m+8/5
plz helppppp

Answers

Answer:

m < 1.4

Step-by-step explanation:

Make [tex]\frac{8}{5}[/tex] a decimal.

1.6

3 > m + 1.6

m < 1.4

Please answer correctly !!!! Will mark brainliest !!!!!!!!!!!!

Answers

Answer:

[tex]x^2+10x+24[/tex]

Step-by-step explanation:

So just multiply the sides:

[tex]x^2+6x+4x+24[/tex]

Which is:

[tex]x^2+10x+24[/tex]

What the answer to this

Answers

Answer:

I think it might be a I don't know for sure though I just need some more points so I can ask a question myself sorry if this didn't help

8. Where will the hour hand of a clock stop if it starts:
a.
from 7 and turns through 1 right angle?
b. from 11 and turns through 3 right angles

can you plz say me the answer​

Answers

Answer:

a. 11

b. 9

Step-by-step explanation:

thats the answer

John has two jobs and earns a total of $2,345 per month. What percent of his gross income does John receive from his second job where he makes $609.70 a month?

Answers

Answer:

10000000.9

Step-by-step explanation:

The lateral area of a right cone which has a base diameter of 4 units and a height of 10 units is:

Answers

Answer:

≈64.08

Step-by-step explanation:

Lateral Area=πr[tex]\sqrt{h^{2} +r^{2} }[/tex]

Radius is 1/2 of the diameter so r=2

Height=10

Substitute:

: [tex]3.141(2)\sqrt{10^{2}+2^{2} } \\6.282\sqrt{100+4} \\6.282\sqrt{104} \\ 64.066.282*10.2\\ 64.076\\\\[/tex]

64.076 rounded = 64.08

please solve y = 3x - 1​

Answers

Step-by-step explanation:

i think question is not complete.

(3x5−2x4−5)−(2x4+x2−10) Subtract the two polynomials

Answers

Answer:

3x^5-4x^4-x^2+5

Step-by-step explanation:

(3x^5−2x^4−5)−(2x^4+x^2−10)

Distribute the minus sign

(3x^5−2x^4−5)−2x^4-x^2+10

Combine like terms

3x^5-4x^4-x^2+5

Hello!

Answer:

[tex]\boxed{ \bf 3x^5~-~4x^4~-~x^2~+~5}[/tex]

__________________________________Explanation:

(3[tex]x^{5}[/tex] - 2[tex]x^{4}[/tex] - 5) - (2[tex]x^{4}[/tex] + x² - 10)

Drop the brackets:

3[tex]x^{5}[/tex] - 2[tex]x^{4}[/tex] - 5 - 2[tex]x^{4}[/tex] - x² + 10

Combine Like Terms:

3[tex]x^{5}[/tex] - 2[tex]x^{4}[/tex] - 2[tex]x^{4}[/tex] - x² - 5 + 10

3[tex]x^{5}[/tex] - 4[tex]x^{4}[/tex] - x² + 5

What is the slope of the graph? slope = -1/3 slope = -3 slope = 3 slope = 1/3

Answers

Answer:

The slope is -3

Step-by-step explanation:

This is so because the line in pointing down, leading to the fact that the slope is negative, and rise over run is 3/1......

Therefore, the slope of the line is -3

To find the slope of the line, first start with a point on the graph.

Let's use the point (0,3) which we call point A.

The other point we will call point B.

Now remember that slope can be found using the ratio rise/run

between any two points that are on that line.

To get from point A to point B along this line, we must

first go down 3 units so we say that our rise is -3.

From there we move 1 unit to the right so our run is 1.

So our slope or rise/run is -3/1 which reduces to -3.

What is the shape of the cross-section formed when a cylinder intersects a
plane as shown in the drawing?

Answers

Answer:

circle,

Step-by-step explanation:

disc in the middle vertically in this case is equal to the bases

The intersection is called an Oval. Hence the correct option is A. An oval in mathematics is a shape just like a circle but with an elongated outline like the shape of an egg.

What is a Cross-section?

A cross-section is a surface, an area that is created or exposed by executing a straight cut across or through a shape.

Cross-sections in technical drawings are used for depicting the internal view of an object that is three-dimensional.

Learn more about Cross-Sections are:

https://brainly.com/question/10511133

pleass more math help

Answers

Can’t answer this question

A line with a slope of -2 crosses the y-axis at (0, 3). The equation of the line is ____

Answers

Answer:

y=-2x+3

Step-by-step explanation:

equation for slope is y=mx+b

the slope is -2 so that equals m

(0,3) is the y-intercept

y=b

3 is the y value making it substitute b making the equation y=-2x+3


Before graduating this year, a senior homeroom was given a survey. Of those surveyed, 24% felt they learned better at home. Of this group, 80% said they plan on taking an online course in college. Of the students who felt they did not learn better at home, 40% said they plan on taking an online course in college

Part A
What is the probability a person who does not plan on taking an online course felt they learned better at home?

A : 2/21
B : 24/125
C : 38/125
D : 19/31
E :None of these

Part B
What is the probability a person who does plan on taking an online course felt they did not learn better at home?

A : 2/21
B : 24/125
C : 38/125
D : 19/31
E : None of these

Answers

Answer:

(A) The correct option is (A).

(B) The correct option is (E).

Step-by-step explanation:

The events can be defined as follows:

X =  students felt they learned better at home

Y = students plan on taking an online course in college

The information provided is:

P (X) = 0.24

P (Y|X) = 0.80

P (Y|X') = 0.40

[tex]P(Y'|X)=1-P(Y|X)\\=1-0.80\\=0.20[/tex]

[tex]P(Y'|X')=1-P(Y|X')\\=1-0.40\\=0.60[/tex]

The Bayes' theorem states that the conditional probability of an event E[tex]_{i}[/tex] given that another event A has already occurred is:

[tex]P(E_{i}|A)=\frac{P(A|E_{i})P(E_{i})}{\sum {P(A|E_{i})P(E_{i})}}[/tex]

(A)

Compute the probability a person who does not plan on taking an online course felt they learned better at home as follows:

Use the Bayes' theorem.

[tex]P(X|Y')=\frac{P(Y'|X)P(X)}{P(Y'|X)P(X)+P(Y'|X')P(X')}[/tex]

              [tex]=\frac{0.20\times 0.24}{(0.20\times 0.24)+(0.60\times 0.76)}\\\\=0.09524\\\\\approx 0.095[/tex]

Thus, the probability a person who does not plan on taking an online course felt they learned better at home is 0.095 or 2/21.

(B)

Compute the probability a person who does plan on taking an online course felt they did not learn better at home as follows:

[tex]P(X'|Y')=1-P(X|Y')\\=1-0.095\\=0.905[/tex]

Thus, the probability a person who does plan on taking an online course felt they did not learn better at home is 0.905.

Other Questions
3x/3=? I WILL MARK U BRAINLIST!!!!! show that x^2-8x+20 can be written in the form (x-a)^2+a. Where a is an integer Overestimate the grocery bill with items that cost: $8.56, $5.34, $34.25, and $6.80.A. $55.00B. $57.00 i need help on this. ive been struggling. TwTlateral areasurface areavolume Examine the following diagram:FederalistsWanted a strongcentral governmentto share powerwith the statesSupportedratificationAnti-Federalists?OpposedratificationDiagram copyright Discovery EducationWhich phrase completes the diagram?Worried about abuses of individual rightsObjected to separation of powersOpposed the institution of slaveryWanted a more unified national system 1. see picture. please include equilibrium and excess demand. 2. based upon the graph you created, what price should bonds gym charge for memberships in order to reach equilibrium? 3. if bonds gym raises prices to your recommended price, how will this affect demand for their product? Thank you! I need a %100 Calculate the sum of the first 50 terms in the sequence.18, 21, 24, 27, ... If the mass of an object is 100kg and the acceleration due to gravity is 9.8m/s^2 what is the weight of the object what did richard nixon good things do during his presidency Why do people care about being popularwhy do others not? Do you yourself value popularity or being well-liked? Cite evidence from this text and your own experience in your answer.. PLEASE HELP ME, it's urgent! 20 POINTS! --Philosophy exists in all religions. Do you agree with this statement? Explain why/why not. How many solutions does the following system of equations have?2y= 5x+4y= 3x+2. TwoB. OneC. ZeroD. Infinitely many Calculate the molar mass of a vapor that has a density of 7.125 g/L at 13 C and 741 torr . I ____________ (work) as a teacher when I first __________ (meet) your mother homograph ground and fall Use the given measurements on the triangle to write the following trig ratios and answer the question that follows Please help! A botanist has discovered a new plant species and is trying to classify the plant. Its seed has one cotyledon, it has six flowerparts, its leaves have parallel veins, its stem is herbaceous, and the vascular bundles in its stem are scattered throughout thestem. Which best classifies this plant?O gymnosperm and dicotO gymnosperm and monocotO angiosperm and monocotO angiosperm and dicot A boy sent a telegram which contains 36 words to his mother if the first 12 words cost $20 and each additional word cost 2$ per word how much did he pay ? Refer to the article "Making Money" in your Money, Money, Money magazine for a complete version of this text. Based on the information in the "Paper Money" section of the article, what is the relationship between the Federal Reserve and the money supply?The organization creates the designs for bills and coins in circulation, ensuring they reflect American values.The organization maintains the five mint companies in the United States where damaged coins are recycled.The Federal Reserve determines when bills should be removed from circulation and when new bills should be produced.The Federal Reserve has two printing companies in the United States where it produces new bills for circulation. What is the length of the hypotenuse of the triangle?